Proving the Empty Intersection of Intervals using Natural Numbers

Click For Summary
The discussion focuses on proving that the intersection of the intervals (0, 1/n) for n from 1 to infinity is empty. Participants note that since 0 is not included in the intervals, any positive number x cannot belong to all intervals as n increases, leading to the conclusion that the intersection is indeed empty. They highlight that if the intervals were closed, such as [0, 1/n], the intersection would contain the point 0. A suggestion is made to refine the proof by demonstrating that for any x > 0, there exists a natural number k such that 1/k is less than x. The conversation emphasizes the importance of interval boundaries in determining the intersection's contents.
cragar
Messages
2,546
Reaction score
3

Homework Statement


Prove that \bigcap_{n=0}^{\inf} (0,\frac{1}{n})=\emptyset

The Attempt at a Solution


since 0 is not included in our interval. eventually I will get to
(0,0) because I could pick a real as close to zero as I wanted and there would be a natural such that \frac{1}{n}<y therefore this intersection is empty.
but if my orginal intersection was [0,1/n] then this would not be empty.
 
Physics news on Phys.org
You have the right idea, and you're correct that if (0,1/n) was replaced by [0,1/n], the intersection would not be empty. It would contain exactly one point: 0.

You could word your proof a bit precisely as follows:

Suppose the intersection is non-empty. Then there exists some x in the intersection:

x \in \bigcap_{n=1}^{\infty} (0, 1/n)

Since x is in the intersection, it means that x must be in every interval of the form (0, 1/n), and this is impossible because...
 
ok thanks for the input
 
you might want to show that if x > 0, there exists k in N with 1/k < x, first.
 
Question: A clock's minute hand has length 4 and its hour hand has length 3. What is the distance between the tips at the moment when it is increasing most rapidly?(Putnam Exam Question) Answer: Making assumption that both the hands moves at constant angular velocities, the answer is ## \sqrt{7} .## But don't you think this assumption is somewhat doubtful and wrong?

Similar threads

  • · Replies 1 ·
Replies
1
Views
2K
  • · Replies 14 ·
Replies
14
Views
2K
  • · Replies 17 ·
Replies
17
Views
8K
  • · Replies 4 ·
Replies
4
Views
1K
  • · Replies 6 ·
Replies
6
Views
2K
  • · Replies 13 ·
Replies
13
Views
2K
  • · Replies 3 ·
Replies
3
Views
2K
Replies
5
Views
2K
  • · Replies 1 ·
Replies
1
Views
2K
  • · Replies 1 ·
Replies
1
Views
1K